Strict convexity and equivalent conditions












3












$begingroup$


Let $f colon mathbb R^n to [0,+infty)$ be a convex, positively 1-homogeneous function, i.e. it holds
$$tag{1}
f(lambda x + (1-lambda)y) le lambda f(x) + (1-lambda) f(y), qquad forall lambda in [0,1], , forall x,y in mathbb R^n
$$

and
$$tag{2}
f(lambda x) = lambda f(x), qquad forall lambda >0, , x in mathbb R^n.
$$



Let me denote by $E_c := { f le c}$ the sub-level set at height $c$. I have been asked to show that Assumption (2) implies that the sub-level sets are homothetic. Indeed I have proved that
$E_c = cE_1$ for every $c>0$.




Now, assuming that $f$ is also of class $C^2(mathbb R^n)$, I have to investigate the validity of the following equivalence:



(A) The set $E_1$ is strictly convex;



(B) There exists $s>0$ such that
$$
nabla^2 f[x] (z,z) ge s vert z - (zcdot x)x vert^2
$$

for every $x,z in mathbb R^n$, being $nabla^2 f[x](z,z) = langle (nabla^2 f)(x) cdot z, z rangle$ the quadratic form associated to the Hessian matrix of $f$ (evaluated at $x$).



Q. Is it true that (A) iff (B)?




I have no idea on how to face the question, and I am looking for some intuition behind condition (B). What is it actually saying? How could it be related to sub-level sets?





ADDENDUM: I think I got some intuition behind condition (B). If I am not wrong that should be a uniform bound (from below) on the Gaussian curvature of the level set ${f=1}$ (assuming that all points are regular, i.e. $vert nabla f(x) vert ne 0$ for every $x in {f=1}$: is this assumption necessary?). This is an easy consequence of the formula contained in this page. Do you agree on this consideration? Now the question becomes: how is it possible to relate a (uniform) bound on the Gaussian curvature of the level set ${f=1}$ with the (strict) convexity of the set ${f le 1}$?



To me this makes sense, because the strict convexity of ${f le 1}$ roughly means that its boundary has no flat parts, and its boundary should be related to the level set - which has curvature bounded from below, i.e. it is well-round...










share|cite|improve this question











$endgroup$








  • 1




    $begingroup$
    Can you explain the notation $ nabla^2 f[x] (z,z)$?
    $endgroup$
    – zhw.
    Jan 1 at 18:44






  • 1




    $begingroup$
    @zhw. It is simply the Hessian matrix of $f$ (computed at $x$) evaluated (as a quadratic form) on the vector $z$. I added clarification in the OP. Thanks for your comment.
    $endgroup$
    – Romeo
    Jan 1 at 19:51










  • $begingroup$
    Strict convexity also is defined in [convex optimization] by Stephen Boyd such that if $$nabla^2 f(x)ge mI$$ for some $m>0$ and for all $xin S$ then $f(x)$ is strictly convex over $S$. Is this definition equivalent to what defined in this question?
    $endgroup$
    – Mostafa Ayaz
    Jan 7 at 12:03










  • $begingroup$
    @MostafaAyaz That is a possible definition of strictly convex function (although it sounds to me more as a "uniform" convexity, at least if $x in mathbb R$). I was more interested in strict convexity of a set but I realize the question is ill posed, I cannot work with $C^2$ functions, otherwise the problem becomes trivial, as LinAlg pointed out.
    $endgroup$
    – Romeo
    Jan 7 at 12:41
















3












$begingroup$


Let $f colon mathbb R^n to [0,+infty)$ be a convex, positively 1-homogeneous function, i.e. it holds
$$tag{1}
f(lambda x + (1-lambda)y) le lambda f(x) + (1-lambda) f(y), qquad forall lambda in [0,1], , forall x,y in mathbb R^n
$$

and
$$tag{2}
f(lambda x) = lambda f(x), qquad forall lambda >0, , x in mathbb R^n.
$$



Let me denote by $E_c := { f le c}$ the sub-level set at height $c$. I have been asked to show that Assumption (2) implies that the sub-level sets are homothetic. Indeed I have proved that
$E_c = cE_1$ for every $c>0$.




Now, assuming that $f$ is also of class $C^2(mathbb R^n)$, I have to investigate the validity of the following equivalence:



(A) The set $E_1$ is strictly convex;



(B) There exists $s>0$ such that
$$
nabla^2 f[x] (z,z) ge s vert z - (zcdot x)x vert^2
$$

for every $x,z in mathbb R^n$, being $nabla^2 f[x](z,z) = langle (nabla^2 f)(x) cdot z, z rangle$ the quadratic form associated to the Hessian matrix of $f$ (evaluated at $x$).



Q. Is it true that (A) iff (B)?




I have no idea on how to face the question, and I am looking for some intuition behind condition (B). What is it actually saying? How could it be related to sub-level sets?





ADDENDUM: I think I got some intuition behind condition (B). If I am not wrong that should be a uniform bound (from below) on the Gaussian curvature of the level set ${f=1}$ (assuming that all points are regular, i.e. $vert nabla f(x) vert ne 0$ for every $x in {f=1}$: is this assumption necessary?). This is an easy consequence of the formula contained in this page. Do you agree on this consideration? Now the question becomes: how is it possible to relate a (uniform) bound on the Gaussian curvature of the level set ${f=1}$ with the (strict) convexity of the set ${f le 1}$?



To me this makes sense, because the strict convexity of ${f le 1}$ roughly means that its boundary has no flat parts, and its boundary should be related to the level set - which has curvature bounded from below, i.e. it is well-round...










share|cite|improve this question











$endgroup$








  • 1




    $begingroup$
    Can you explain the notation $ nabla^2 f[x] (z,z)$?
    $endgroup$
    – zhw.
    Jan 1 at 18:44






  • 1




    $begingroup$
    @zhw. It is simply the Hessian matrix of $f$ (computed at $x$) evaluated (as a quadratic form) on the vector $z$. I added clarification in the OP. Thanks for your comment.
    $endgroup$
    – Romeo
    Jan 1 at 19:51










  • $begingroup$
    Strict convexity also is defined in [convex optimization] by Stephen Boyd such that if $$nabla^2 f(x)ge mI$$ for some $m>0$ and for all $xin S$ then $f(x)$ is strictly convex over $S$. Is this definition equivalent to what defined in this question?
    $endgroup$
    – Mostafa Ayaz
    Jan 7 at 12:03










  • $begingroup$
    @MostafaAyaz That is a possible definition of strictly convex function (although it sounds to me more as a "uniform" convexity, at least if $x in mathbb R$). I was more interested in strict convexity of a set but I realize the question is ill posed, I cannot work with $C^2$ functions, otherwise the problem becomes trivial, as LinAlg pointed out.
    $endgroup$
    – Romeo
    Jan 7 at 12:41














3












3








3


1



$begingroup$


Let $f colon mathbb R^n to [0,+infty)$ be a convex, positively 1-homogeneous function, i.e. it holds
$$tag{1}
f(lambda x + (1-lambda)y) le lambda f(x) + (1-lambda) f(y), qquad forall lambda in [0,1], , forall x,y in mathbb R^n
$$

and
$$tag{2}
f(lambda x) = lambda f(x), qquad forall lambda >0, , x in mathbb R^n.
$$



Let me denote by $E_c := { f le c}$ the sub-level set at height $c$. I have been asked to show that Assumption (2) implies that the sub-level sets are homothetic. Indeed I have proved that
$E_c = cE_1$ for every $c>0$.




Now, assuming that $f$ is also of class $C^2(mathbb R^n)$, I have to investigate the validity of the following equivalence:



(A) The set $E_1$ is strictly convex;



(B) There exists $s>0$ such that
$$
nabla^2 f[x] (z,z) ge s vert z - (zcdot x)x vert^2
$$

for every $x,z in mathbb R^n$, being $nabla^2 f[x](z,z) = langle (nabla^2 f)(x) cdot z, z rangle$ the quadratic form associated to the Hessian matrix of $f$ (evaluated at $x$).



Q. Is it true that (A) iff (B)?




I have no idea on how to face the question, and I am looking for some intuition behind condition (B). What is it actually saying? How could it be related to sub-level sets?





ADDENDUM: I think I got some intuition behind condition (B). If I am not wrong that should be a uniform bound (from below) on the Gaussian curvature of the level set ${f=1}$ (assuming that all points are regular, i.e. $vert nabla f(x) vert ne 0$ for every $x in {f=1}$: is this assumption necessary?). This is an easy consequence of the formula contained in this page. Do you agree on this consideration? Now the question becomes: how is it possible to relate a (uniform) bound on the Gaussian curvature of the level set ${f=1}$ with the (strict) convexity of the set ${f le 1}$?



To me this makes sense, because the strict convexity of ${f le 1}$ roughly means that its boundary has no flat parts, and its boundary should be related to the level set - which has curvature bounded from below, i.e. it is well-round...










share|cite|improve this question











$endgroup$




Let $f colon mathbb R^n to [0,+infty)$ be a convex, positively 1-homogeneous function, i.e. it holds
$$tag{1}
f(lambda x + (1-lambda)y) le lambda f(x) + (1-lambda) f(y), qquad forall lambda in [0,1], , forall x,y in mathbb R^n
$$

and
$$tag{2}
f(lambda x) = lambda f(x), qquad forall lambda >0, , x in mathbb R^n.
$$



Let me denote by $E_c := { f le c}$ the sub-level set at height $c$. I have been asked to show that Assumption (2) implies that the sub-level sets are homothetic. Indeed I have proved that
$E_c = cE_1$ for every $c>0$.




Now, assuming that $f$ is also of class $C^2(mathbb R^n)$, I have to investigate the validity of the following equivalence:



(A) The set $E_1$ is strictly convex;



(B) There exists $s>0$ such that
$$
nabla^2 f[x] (z,z) ge s vert z - (zcdot x)x vert^2
$$

for every $x,z in mathbb R^n$, being $nabla^2 f[x](z,z) = langle (nabla^2 f)(x) cdot z, z rangle$ the quadratic form associated to the Hessian matrix of $f$ (evaluated at $x$).



Q. Is it true that (A) iff (B)?




I have no idea on how to face the question, and I am looking for some intuition behind condition (B). What is it actually saying? How could it be related to sub-level sets?





ADDENDUM: I think I got some intuition behind condition (B). If I am not wrong that should be a uniform bound (from below) on the Gaussian curvature of the level set ${f=1}$ (assuming that all points are regular, i.e. $vert nabla f(x) vert ne 0$ for every $x in {f=1}$: is this assumption necessary?). This is an easy consequence of the formula contained in this page. Do you agree on this consideration? Now the question becomes: how is it possible to relate a (uniform) bound on the Gaussian curvature of the level set ${f=1}$ with the (strict) convexity of the set ${f le 1}$?



To me this makes sense, because the strict convexity of ${f le 1}$ roughly means that its boundary has no flat parts, and its boundary should be related to the level set - which has curvature bounded from below, i.e. it is well-round...







real-analysis functions convex-analysis






share|cite|improve this question















share|cite|improve this question













share|cite|improve this question




share|cite|improve this question








edited Jan 1 at 19:50







Romeo

















asked Dec 30 '18 at 16:28









RomeoRomeo

2,99321148




2,99321148








  • 1




    $begingroup$
    Can you explain the notation $ nabla^2 f[x] (z,z)$?
    $endgroup$
    – zhw.
    Jan 1 at 18:44






  • 1




    $begingroup$
    @zhw. It is simply the Hessian matrix of $f$ (computed at $x$) evaluated (as a quadratic form) on the vector $z$. I added clarification in the OP. Thanks for your comment.
    $endgroup$
    – Romeo
    Jan 1 at 19:51










  • $begingroup$
    Strict convexity also is defined in [convex optimization] by Stephen Boyd such that if $$nabla^2 f(x)ge mI$$ for some $m>0$ and for all $xin S$ then $f(x)$ is strictly convex over $S$. Is this definition equivalent to what defined in this question?
    $endgroup$
    – Mostafa Ayaz
    Jan 7 at 12:03










  • $begingroup$
    @MostafaAyaz That is a possible definition of strictly convex function (although it sounds to me more as a "uniform" convexity, at least if $x in mathbb R$). I was more interested in strict convexity of a set but I realize the question is ill posed, I cannot work with $C^2$ functions, otherwise the problem becomes trivial, as LinAlg pointed out.
    $endgroup$
    – Romeo
    Jan 7 at 12:41














  • 1




    $begingroup$
    Can you explain the notation $ nabla^2 f[x] (z,z)$?
    $endgroup$
    – zhw.
    Jan 1 at 18:44






  • 1




    $begingroup$
    @zhw. It is simply the Hessian matrix of $f$ (computed at $x$) evaluated (as a quadratic form) on the vector $z$. I added clarification in the OP. Thanks for your comment.
    $endgroup$
    – Romeo
    Jan 1 at 19:51










  • $begingroup$
    Strict convexity also is defined in [convex optimization] by Stephen Boyd such that if $$nabla^2 f(x)ge mI$$ for some $m>0$ and for all $xin S$ then $f(x)$ is strictly convex over $S$. Is this definition equivalent to what defined in this question?
    $endgroup$
    – Mostafa Ayaz
    Jan 7 at 12:03










  • $begingroup$
    @MostafaAyaz That is a possible definition of strictly convex function (although it sounds to me more as a "uniform" convexity, at least if $x in mathbb R$). I was more interested in strict convexity of a set but I realize the question is ill posed, I cannot work with $C^2$ functions, otherwise the problem becomes trivial, as LinAlg pointed out.
    $endgroup$
    – Romeo
    Jan 7 at 12:41








1




1




$begingroup$
Can you explain the notation $ nabla^2 f[x] (z,z)$?
$endgroup$
– zhw.
Jan 1 at 18:44




$begingroup$
Can you explain the notation $ nabla^2 f[x] (z,z)$?
$endgroup$
– zhw.
Jan 1 at 18:44




1




1




$begingroup$
@zhw. It is simply the Hessian matrix of $f$ (computed at $x$) evaluated (as a quadratic form) on the vector $z$. I added clarification in the OP. Thanks for your comment.
$endgroup$
– Romeo
Jan 1 at 19:51




$begingroup$
@zhw. It is simply the Hessian matrix of $f$ (computed at $x$) evaluated (as a quadratic form) on the vector $z$. I added clarification in the OP. Thanks for your comment.
$endgroup$
– Romeo
Jan 1 at 19:51












$begingroup$
Strict convexity also is defined in [convex optimization] by Stephen Boyd such that if $$nabla^2 f(x)ge mI$$ for some $m>0$ and for all $xin S$ then $f(x)$ is strictly convex over $S$. Is this definition equivalent to what defined in this question?
$endgroup$
– Mostafa Ayaz
Jan 7 at 12:03




$begingroup$
Strict convexity also is defined in [convex optimization] by Stephen Boyd such that if $$nabla^2 f(x)ge mI$$ for some $m>0$ and for all $xin S$ then $f(x)$ is strictly convex over $S$. Is this definition equivalent to what defined in this question?
$endgroup$
– Mostafa Ayaz
Jan 7 at 12:03












$begingroup$
@MostafaAyaz That is a possible definition of strictly convex function (although it sounds to me more as a "uniform" convexity, at least if $x in mathbb R$). I was more interested in strict convexity of a set but I realize the question is ill posed, I cannot work with $C^2$ functions, otherwise the problem becomes trivial, as LinAlg pointed out.
$endgroup$
– Romeo
Jan 7 at 12:41




$begingroup$
@MostafaAyaz That is a possible definition of strictly convex function (although it sounds to me more as a "uniform" convexity, at least if $x in mathbb R$). I was more interested in strict convexity of a set but I realize the question is ill posed, I cannot work with $C^2$ functions, otherwise the problem becomes trivial, as LinAlg pointed out.
$endgroup$
– Romeo
Jan 7 at 12:41










2 Answers
2






active

oldest

votes


















1





+200







$begingroup$

The only function I can think of that satisfies all your conditions (convex, range $[0,infty)$, positively 1-homogeneous, $C^2$) is the function that always takes the value $0$.



You cannot have a positively 1-homogeneous function for which $E_1$ is strictly convex: the function is linear on the line segment ${ lambda x : f(lambda x) leq 1 } subseteq E_1$ (the line segment cannot be a single point or an empty set).



Now let's look at condition B. Since (2) holds for all $x$, the derivatives have to be equal as well: $lambdanabla f(lambda x) = lambda nabla f(x)$. It follows that $nabla f(lambda x)$ is the same for all $lambda$, so the second derivative at $0$ in the direction $z$ is $0$: $nabla^2f[0](z,z)=0$.



As neither condition (A) nor condition (B) can ever hold, they are equivalent.






share|cite|improve this answer









$endgroup$













  • $begingroup$
    First of all, thank you very much for your very interesting answer. May I ask some further questions? In the first line you wrote: convex, range $[0,+infty)$, pos 1-hom and $C^2$ imply identically vanishing. Can you provide a short proof of this elementary fact? I am not able to see it. Concerning assumption $A$, I see your point and I realized the question is ill-posed. What I actually have is that the boundary of the set ${fle 1}$ does not contain straight segments. Concerning assumption (B) I do not see your conclusion (the hessian vanishes hence...?). Thank you very much for your help.
    $endgroup$
    – Romeo
    Jan 4 at 9:36










  • $begingroup$
    @Romeo First line: I do not have a proof but it's merely a conjecture ("the only function I can think of"). Part A: are you sure? Then $f(x)=0$ is a counterexample. Part B: the Hessian vanishes at 0 for all $z$, so the Hessian cannot be bounded below by $s|z|^2$.
    $endgroup$
    – LinAlg
    Jan 4 at 14:43












  • $begingroup$
    Thanks a lot for your valuable help. Thanks to your post I understood I have to reconsider everything, your conjecture in particular (later proved by Gio67) was a real eye-opener. Thanks a lot.
    $endgroup$
    – Romeo
    Jan 7 at 12:45










  • $begingroup$
    Yet, I realized - along the suggestion of Gio67 - that it is possible to formulate the same question asking only $f in C^2(mathbb R^n setminus {0})$ (and condition (B) holding for every $x ne 0$). Suppose $f$ is not identically vanishing, too. Is it true that the set $E_1$ does not contain straight segments iff (B) holds? Thank you very much.
    $endgroup$
    – Romeo
    Jan 10 at 9:28










  • $begingroup$
    @Romeo I'd need to think about that and do not have time for that right now. You should edit your question and unaccept my answer or post a new question.
    $endgroup$
    – LinAlg
    Jan 11 at 9:38



















2












$begingroup$

I think you need to assume $fin C^{2}(mathbb{R}^{n}setminus{0})$ and not
$fin C^{2}(mathbb{R}^{n})$. Otherwise, if $t>0$,
$$
frac{f(te_{i})-f(0)}{t}=frac{tfleft( e_{i}right) }{t}=f(e_{i}),
$$

while if $t<0$,
$$
frac{f(te_{i})-f(0)}{t}=-frac{tfleft( -e_{i}right) }{t}=-f(-e_{i}),
$$

and so if $frac{partial f}{partial x_{i}}(0)$ exists, then necessarily,
$f(e_{i})=-f(-e_{i})$, which is only possible if $f(e_{i})=f(-e_{i})=0$, since
$fgeq0$. But then by convexity, writing
$$
x=frac{1}{n}sum_{i=1}^{n}nx_{i}e_{i}%
$$

you get
$$
0leq f(x)leqfrac{1}{n}sum_{i=1}^{n}f(nx_{i}e_{i})=frac{1}{n}sum
_{i=1}^{n}n|x_{i}|f((operatorname*{sgn}x_{i})e_{i})=0,
$$

which implies that $f=0$.



So LinAlg's conjecture is true.



Note also that if a function $fin C^{1}(mathbb{R}^{n}setminus{0})$ is
positively homogeneous of degree one, then its partial derivatives
$frac{partial f}{partial x_{i}}$ are positively homogeneous of degree zero,
so $frac{partial f}{partial x_{i}}(tx)=frac{partial f}{partial x_{i}%
}(x)$
for all $t>0$.






share|cite|improve this answer









$endgroup$













  • $begingroup$
    Thank you for proving my conjecture.
    $endgroup$
    – LinAlg
    Jan 6 at 20:03










  • $begingroup$
    Thanks for your interesting post, providing a proof of LinAlg's conjecture. I hope you do not mind, I will accept his answer and give him the bounty. (Un)fortunately thanks to his answer (as well as your post) I realized my question is ill-posed and I need to review everything. Thank you very much for your time and help. +1, of course.
    $endgroup$
    – Romeo
    Jan 7 at 12:38










  • $begingroup$
    Thanks to what you wrote in the first line, I realized it is still meaningful to formulate the same question asking only $f in C^2(mathbb R^n setminus {0})$ (and condition (B) holding for every $x ne 0$). Suppose f is not identically vanishing, too. Is it true that the set $E_1$ does not contain straight segments iff (B) holds? Thank you very much again.
    $endgroup$
    – Romeo
    Jan 10 at 9:39













Your Answer





StackExchange.ifUsing("editor", function () {
return StackExchange.using("mathjaxEditing", function () {
StackExchange.MarkdownEditor.creationCallbacks.add(function (editor, postfix) {
StackExchange.mathjaxEditing.prepareWmdForMathJax(editor, postfix, [["$", "$"], ["\\(","\\)"]]);
});
});
}, "mathjax-editing");

StackExchange.ready(function() {
var channelOptions = {
tags: "".split(" "),
id: "69"
};
initTagRenderer("".split(" "), "".split(" "), channelOptions);

StackExchange.using("externalEditor", function() {
// Have to fire editor after snippets, if snippets enabled
if (StackExchange.settings.snippets.snippetsEnabled) {
StackExchange.using("snippets", function() {
createEditor();
});
}
else {
createEditor();
}
});

function createEditor() {
StackExchange.prepareEditor({
heartbeatType: 'answer',
autoActivateHeartbeat: false,
convertImagesToLinks: true,
noModals: true,
showLowRepImageUploadWarning: true,
reputationToPostImages: 10,
bindNavPrevention: true,
postfix: "",
imageUploader: {
brandingHtml: "Powered by u003ca class="icon-imgur-white" href="https://imgur.com/"u003eu003c/au003e",
contentPolicyHtml: "User contributions licensed under u003ca href="https://creativecommons.org/licenses/by-sa/3.0/"u003ecc by-sa 3.0 with attribution requiredu003c/au003e u003ca href="https://stackoverflow.com/legal/content-policy"u003e(content policy)u003c/au003e",
allowUrls: true
},
noCode: true, onDemand: true,
discardSelector: ".discard-answer"
,immediatelyShowMarkdownHelp:true
});


}
});














draft saved

draft discarded


















StackExchange.ready(
function () {
StackExchange.openid.initPostLogin('.new-post-login', 'https%3a%2f%2fmath.stackexchange.com%2fquestions%2f3056984%2fstrict-convexity-and-equivalent-conditions%23new-answer', 'question_page');
}
);

Post as a guest















Required, but never shown

























2 Answers
2






active

oldest

votes








2 Answers
2






active

oldest

votes









active

oldest

votes






active

oldest

votes









1





+200







$begingroup$

The only function I can think of that satisfies all your conditions (convex, range $[0,infty)$, positively 1-homogeneous, $C^2$) is the function that always takes the value $0$.



You cannot have a positively 1-homogeneous function for which $E_1$ is strictly convex: the function is linear on the line segment ${ lambda x : f(lambda x) leq 1 } subseteq E_1$ (the line segment cannot be a single point or an empty set).



Now let's look at condition B. Since (2) holds for all $x$, the derivatives have to be equal as well: $lambdanabla f(lambda x) = lambda nabla f(x)$. It follows that $nabla f(lambda x)$ is the same for all $lambda$, so the second derivative at $0$ in the direction $z$ is $0$: $nabla^2f[0](z,z)=0$.



As neither condition (A) nor condition (B) can ever hold, they are equivalent.






share|cite|improve this answer









$endgroup$













  • $begingroup$
    First of all, thank you very much for your very interesting answer. May I ask some further questions? In the first line you wrote: convex, range $[0,+infty)$, pos 1-hom and $C^2$ imply identically vanishing. Can you provide a short proof of this elementary fact? I am not able to see it. Concerning assumption $A$, I see your point and I realized the question is ill-posed. What I actually have is that the boundary of the set ${fle 1}$ does not contain straight segments. Concerning assumption (B) I do not see your conclusion (the hessian vanishes hence...?). Thank you very much for your help.
    $endgroup$
    – Romeo
    Jan 4 at 9:36










  • $begingroup$
    @Romeo First line: I do not have a proof but it's merely a conjecture ("the only function I can think of"). Part A: are you sure? Then $f(x)=0$ is a counterexample. Part B: the Hessian vanishes at 0 for all $z$, so the Hessian cannot be bounded below by $s|z|^2$.
    $endgroup$
    – LinAlg
    Jan 4 at 14:43












  • $begingroup$
    Thanks a lot for your valuable help. Thanks to your post I understood I have to reconsider everything, your conjecture in particular (later proved by Gio67) was a real eye-opener. Thanks a lot.
    $endgroup$
    – Romeo
    Jan 7 at 12:45










  • $begingroup$
    Yet, I realized - along the suggestion of Gio67 - that it is possible to formulate the same question asking only $f in C^2(mathbb R^n setminus {0})$ (and condition (B) holding for every $x ne 0$). Suppose $f$ is not identically vanishing, too. Is it true that the set $E_1$ does not contain straight segments iff (B) holds? Thank you very much.
    $endgroup$
    – Romeo
    Jan 10 at 9:28










  • $begingroup$
    @Romeo I'd need to think about that and do not have time for that right now. You should edit your question and unaccept my answer or post a new question.
    $endgroup$
    – LinAlg
    Jan 11 at 9:38
















1





+200







$begingroup$

The only function I can think of that satisfies all your conditions (convex, range $[0,infty)$, positively 1-homogeneous, $C^2$) is the function that always takes the value $0$.



You cannot have a positively 1-homogeneous function for which $E_1$ is strictly convex: the function is linear on the line segment ${ lambda x : f(lambda x) leq 1 } subseteq E_1$ (the line segment cannot be a single point or an empty set).



Now let's look at condition B. Since (2) holds for all $x$, the derivatives have to be equal as well: $lambdanabla f(lambda x) = lambda nabla f(x)$. It follows that $nabla f(lambda x)$ is the same for all $lambda$, so the second derivative at $0$ in the direction $z$ is $0$: $nabla^2f[0](z,z)=0$.



As neither condition (A) nor condition (B) can ever hold, they are equivalent.






share|cite|improve this answer









$endgroup$













  • $begingroup$
    First of all, thank you very much for your very interesting answer. May I ask some further questions? In the first line you wrote: convex, range $[0,+infty)$, pos 1-hom and $C^2$ imply identically vanishing. Can you provide a short proof of this elementary fact? I am not able to see it. Concerning assumption $A$, I see your point and I realized the question is ill-posed. What I actually have is that the boundary of the set ${fle 1}$ does not contain straight segments. Concerning assumption (B) I do not see your conclusion (the hessian vanishes hence...?). Thank you very much for your help.
    $endgroup$
    – Romeo
    Jan 4 at 9:36










  • $begingroup$
    @Romeo First line: I do not have a proof but it's merely a conjecture ("the only function I can think of"). Part A: are you sure? Then $f(x)=0$ is a counterexample. Part B: the Hessian vanishes at 0 for all $z$, so the Hessian cannot be bounded below by $s|z|^2$.
    $endgroup$
    – LinAlg
    Jan 4 at 14:43












  • $begingroup$
    Thanks a lot for your valuable help. Thanks to your post I understood I have to reconsider everything, your conjecture in particular (later proved by Gio67) was a real eye-opener. Thanks a lot.
    $endgroup$
    – Romeo
    Jan 7 at 12:45










  • $begingroup$
    Yet, I realized - along the suggestion of Gio67 - that it is possible to formulate the same question asking only $f in C^2(mathbb R^n setminus {0})$ (and condition (B) holding for every $x ne 0$). Suppose $f$ is not identically vanishing, too. Is it true that the set $E_1$ does not contain straight segments iff (B) holds? Thank you very much.
    $endgroup$
    – Romeo
    Jan 10 at 9:28










  • $begingroup$
    @Romeo I'd need to think about that and do not have time for that right now. You should edit your question and unaccept my answer or post a new question.
    $endgroup$
    – LinAlg
    Jan 11 at 9:38














1





+200







1





+200



1




+200



$begingroup$

The only function I can think of that satisfies all your conditions (convex, range $[0,infty)$, positively 1-homogeneous, $C^2$) is the function that always takes the value $0$.



You cannot have a positively 1-homogeneous function for which $E_1$ is strictly convex: the function is linear on the line segment ${ lambda x : f(lambda x) leq 1 } subseteq E_1$ (the line segment cannot be a single point or an empty set).



Now let's look at condition B. Since (2) holds for all $x$, the derivatives have to be equal as well: $lambdanabla f(lambda x) = lambda nabla f(x)$. It follows that $nabla f(lambda x)$ is the same for all $lambda$, so the second derivative at $0$ in the direction $z$ is $0$: $nabla^2f[0](z,z)=0$.



As neither condition (A) nor condition (B) can ever hold, they are equivalent.






share|cite|improve this answer









$endgroup$



The only function I can think of that satisfies all your conditions (convex, range $[0,infty)$, positively 1-homogeneous, $C^2$) is the function that always takes the value $0$.



You cannot have a positively 1-homogeneous function for which $E_1$ is strictly convex: the function is linear on the line segment ${ lambda x : f(lambda x) leq 1 } subseteq E_1$ (the line segment cannot be a single point or an empty set).



Now let's look at condition B. Since (2) holds for all $x$, the derivatives have to be equal as well: $lambdanabla f(lambda x) = lambda nabla f(x)$. It follows that $nabla f(lambda x)$ is the same for all $lambda$, so the second derivative at $0$ in the direction $z$ is $0$: $nabla^2f[0](z,z)=0$.



As neither condition (A) nor condition (B) can ever hold, they are equivalent.







share|cite|improve this answer












share|cite|improve this answer



share|cite|improve this answer










answered Jan 3 at 17:21









LinAlgLinAlg

8,8511521




8,8511521












  • $begingroup$
    First of all, thank you very much for your very interesting answer. May I ask some further questions? In the first line you wrote: convex, range $[0,+infty)$, pos 1-hom and $C^2$ imply identically vanishing. Can you provide a short proof of this elementary fact? I am not able to see it. Concerning assumption $A$, I see your point and I realized the question is ill-posed. What I actually have is that the boundary of the set ${fle 1}$ does not contain straight segments. Concerning assumption (B) I do not see your conclusion (the hessian vanishes hence...?). Thank you very much for your help.
    $endgroup$
    – Romeo
    Jan 4 at 9:36










  • $begingroup$
    @Romeo First line: I do not have a proof but it's merely a conjecture ("the only function I can think of"). Part A: are you sure? Then $f(x)=0$ is a counterexample. Part B: the Hessian vanishes at 0 for all $z$, so the Hessian cannot be bounded below by $s|z|^2$.
    $endgroup$
    – LinAlg
    Jan 4 at 14:43












  • $begingroup$
    Thanks a lot for your valuable help. Thanks to your post I understood I have to reconsider everything, your conjecture in particular (later proved by Gio67) was a real eye-opener. Thanks a lot.
    $endgroup$
    – Romeo
    Jan 7 at 12:45










  • $begingroup$
    Yet, I realized - along the suggestion of Gio67 - that it is possible to formulate the same question asking only $f in C^2(mathbb R^n setminus {0})$ (and condition (B) holding for every $x ne 0$). Suppose $f$ is not identically vanishing, too. Is it true that the set $E_1$ does not contain straight segments iff (B) holds? Thank you very much.
    $endgroup$
    – Romeo
    Jan 10 at 9:28










  • $begingroup$
    @Romeo I'd need to think about that and do not have time for that right now. You should edit your question and unaccept my answer or post a new question.
    $endgroup$
    – LinAlg
    Jan 11 at 9:38


















  • $begingroup$
    First of all, thank you very much for your very interesting answer. May I ask some further questions? In the first line you wrote: convex, range $[0,+infty)$, pos 1-hom and $C^2$ imply identically vanishing. Can you provide a short proof of this elementary fact? I am not able to see it. Concerning assumption $A$, I see your point and I realized the question is ill-posed. What I actually have is that the boundary of the set ${fle 1}$ does not contain straight segments. Concerning assumption (B) I do not see your conclusion (the hessian vanishes hence...?). Thank you very much for your help.
    $endgroup$
    – Romeo
    Jan 4 at 9:36










  • $begingroup$
    @Romeo First line: I do not have a proof but it's merely a conjecture ("the only function I can think of"). Part A: are you sure? Then $f(x)=0$ is a counterexample. Part B: the Hessian vanishes at 0 for all $z$, so the Hessian cannot be bounded below by $s|z|^2$.
    $endgroup$
    – LinAlg
    Jan 4 at 14:43












  • $begingroup$
    Thanks a lot for your valuable help. Thanks to your post I understood I have to reconsider everything, your conjecture in particular (later proved by Gio67) was a real eye-opener. Thanks a lot.
    $endgroup$
    – Romeo
    Jan 7 at 12:45










  • $begingroup$
    Yet, I realized - along the suggestion of Gio67 - that it is possible to formulate the same question asking only $f in C^2(mathbb R^n setminus {0})$ (and condition (B) holding for every $x ne 0$). Suppose $f$ is not identically vanishing, too. Is it true that the set $E_1$ does not contain straight segments iff (B) holds? Thank you very much.
    $endgroup$
    – Romeo
    Jan 10 at 9:28










  • $begingroup$
    @Romeo I'd need to think about that and do not have time for that right now. You should edit your question and unaccept my answer or post a new question.
    $endgroup$
    – LinAlg
    Jan 11 at 9:38
















$begingroup$
First of all, thank you very much for your very interesting answer. May I ask some further questions? In the first line you wrote: convex, range $[0,+infty)$, pos 1-hom and $C^2$ imply identically vanishing. Can you provide a short proof of this elementary fact? I am not able to see it. Concerning assumption $A$, I see your point and I realized the question is ill-posed. What I actually have is that the boundary of the set ${fle 1}$ does not contain straight segments. Concerning assumption (B) I do not see your conclusion (the hessian vanishes hence...?). Thank you very much for your help.
$endgroup$
– Romeo
Jan 4 at 9:36




$begingroup$
First of all, thank you very much for your very interesting answer. May I ask some further questions? In the first line you wrote: convex, range $[0,+infty)$, pos 1-hom and $C^2$ imply identically vanishing. Can you provide a short proof of this elementary fact? I am not able to see it. Concerning assumption $A$, I see your point and I realized the question is ill-posed. What I actually have is that the boundary of the set ${fle 1}$ does not contain straight segments. Concerning assumption (B) I do not see your conclusion (the hessian vanishes hence...?). Thank you very much for your help.
$endgroup$
– Romeo
Jan 4 at 9:36












$begingroup$
@Romeo First line: I do not have a proof but it's merely a conjecture ("the only function I can think of"). Part A: are you sure? Then $f(x)=0$ is a counterexample. Part B: the Hessian vanishes at 0 for all $z$, so the Hessian cannot be bounded below by $s|z|^2$.
$endgroup$
– LinAlg
Jan 4 at 14:43






$begingroup$
@Romeo First line: I do not have a proof but it's merely a conjecture ("the only function I can think of"). Part A: are you sure? Then $f(x)=0$ is a counterexample. Part B: the Hessian vanishes at 0 for all $z$, so the Hessian cannot be bounded below by $s|z|^2$.
$endgroup$
– LinAlg
Jan 4 at 14:43














$begingroup$
Thanks a lot for your valuable help. Thanks to your post I understood I have to reconsider everything, your conjecture in particular (later proved by Gio67) was a real eye-opener. Thanks a lot.
$endgroup$
– Romeo
Jan 7 at 12:45




$begingroup$
Thanks a lot for your valuable help. Thanks to your post I understood I have to reconsider everything, your conjecture in particular (later proved by Gio67) was a real eye-opener. Thanks a lot.
$endgroup$
– Romeo
Jan 7 at 12:45












$begingroup$
Yet, I realized - along the suggestion of Gio67 - that it is possible to formulate the same question asking only $f in C^2(mathbb R^n setminus {0})$ (and condition (B) holding for every $x ne 0$). Suppose $f$ is not identically vanishing, too. Is it true that the set $E_1$ does not contain straight segments iff (B) holds? Thank you very much.
$endgroup$
– Romeo
Jan 10 at 9:28




$begingroup$
Yet, I realized - along the suggestion of Gio67 - that it is possible to formulate the same question asking only $f in C^2(mathbb R^n setminus {0})$ (and condition (B) holding for every $x ne 0$). Suppose $f$ is not identically vanishing, too. Is it true that the set $E_1$ does not contain straight segments iff (B) holds? Thank you very much.
$endgroup$
– Romeo
Jan 10 at 9:28












$begingroup$
@Romeo I'd need to think about that and do not have time for that right now. You should edit your question and unaccept my answer or post a new question.
$endgroup$
– LinAlg
Jan 11 at 9:38




$begingroup$
@Romeo I'd need to think about that and do not have time for that right now. You should edit your question and unaccept my answer or post a new question.
$endgroup$
– LinAlg
Jan 11 at 9:38











2












$begingroup$

I think you need to assume $fin C^{2}(mathbb{R}^{n}setminus{0})$ and not
$fin C^{2}(mathbb{R}^{n})$. Otherwise, if $t>0$,
$$
frac{f(te_{i})-f(0)}{t}=frac{tfleft( e_{i}right) }{t}=f(e_{i}),
$$

while if $t<0$,
$$
frac{f(te_{i})-f(0)}{t}=-frac{tfleft( -e_{i}right) }{t}=-f(-e_{i}),
$$

and so if $frac{partial f}{partial x_{i}}(0)$ exists, then necessarily,
$f(e_{i})=-f(-e_{i})$, which is only possible if $f(e_{i})=f(-e_{i})=0$, since
$fgeq0$. But then by convexity, writing
$$
x=frac{1}{n}sum_{i=1}^{n}nx_{i}e_{i}%
$$

you get
$$
0leq f(x)leqfrac{1}{n}sum_{i=1}^{n}f(nx_{i}e_{i})=frac{1}{n}sum
_{i=1}^{n}n|x_{i}|f((operatorname*{sgn}x_{i})e_{i})=0,
$$

which implies that $f=0$.



So LinAlg's conjecture is true.



Note also that if a function $fin C^{1}(mathbb{R}^{n}setminus{0})$ is
positively homogeneous of degree one, then its partial derivatives
$frac{partial f}{partial x_{i}}$ are positively homogeneous of degree zero,
so $frac{partial f}{partial x_{i}}(tx)=frac{partial f}{partial x_{i}%
}(x)$
for all $t>0$.






share|cite|improve this answer









$endgroup$













  • $begingroup$
    Thank you for proving my conjecture.
    $endgroup$
    – LinAlg
    Jan 6 at 20:03










  • $begingroup$
    Thanks for your interesting post, providing a proof of LinAlg's conjecture. I hope you do not mind, I will accept his answer and give him the bounty. (Un)fortunately thanks to his answer (as well as your post) I realized my question is ill-posed and I need to review everything. Thank you very much for your time and help. +1, of course.
    $endgroup$
    – Romeo
    Jan 7 at 12:38










  • $begingroup$
    Thanks to what you wrote in the first line, I realized it is still meaningful to formulate the same question asking only $f in C^2(mathbb R^n setminus {0})$ (and condition (B) holding for every $x ne 0$). Suppose f is not identically vanishing, too. Is it true that the set $E_1$ does not contain straight segments iff (B) holds? Thank you very much again.
    $endgroup$
    – Romeo
    Jan 10 at 9:39


















2












$begingroup$

I think you need to assume $fin C^{2}(mathbb{R}^{n}setminus{0})$ and not
$fin C^{2}(mathbb{R}^{n})$. Otherwise, if $t>0$,
$$
frac{f(te_{i})-f(0)}{t}=frac{tfleft( e_{i}right) }{t}=f(e_{i}),
$$

while if $t<0$,
$$
frac{f(te_{i})-f(0)}{t}=-frac{tfleft( -e_{i}right) }{t}=-f(-e_{i}),
$$

and so if $frac{partial f}{partial x_{i}}(0)$ exists, then necessarily,
$f(e_{i})=-f(-e_{i})$, which is only possible if $f(e_{i})=f(-e_{i})=0$, since
$fgeq0$. But then by convexity, writing
$$
x=frac{1}{n}sum_{i=1}^{n}nx_{i}e_{i}%
$$

you get
$$
0leq f(x)leqfrac{1}{n}sum_{i=1}^{n}f(nx_{i}e_{i})=frac{1}{n}sum
_{i=1}^{n}n|x_{i}|f((operatorname*{sgn}x_{i})e_{i})=0,
$$

which implies that $f=0$.



So LinAlg's conjecture is true.



Note also that if a function $fin C^{1}(mathbb{R}^{n}setminus{0})$ is
positively homogeneous of degree one, then its partial derivatives
$frac{partial f}{partial x_{i}}$ are positively homogeneous of degree zero,
so $frac{partial f}{partial x_{i}}(tx)=frac{partial f}{partial x_{i}%
}(x)$
for all $t>0$.






share|cite|improve this answer









$endgroup$













  • $begingroup$
    Thank you for proving my conjecture.
    $endgroup$
    – LinAlg
    Jan 6 at 20:03










  • $begingroup$
    Thanks for your interesting post, providing a proof of LinAlg's conjecture. I hope you do not mind, I will accept his answer and give him the bounty. (Un)fortunately thanks to his answer (as well as your post) I realized my question is ill-posed and I need to review everything. Thank you very much for your time and help. +1, of course.
    $endgroup$
    – Romeo
    Jan 7 at 12:38










  • $begingroup$
    Thanks to what you wrote in the first line, I realized it is still meaningful to formulate the same question asking only $f in C^2(mathbb R^n setminus {0})$ (and condition (B) holding for every $x ne 0$). Suppose f is not identically vanishing, too. Is it true that the set $E_1$ does not contain straight segments iff (B) holds? Thank you very much again.
    $endgroup$
    – Romeo
    Jan 10 at 9:39
















2












2








2





$begingroup$

I think you need to assume $fin C^{2}(mathbb{R}^{n}setminus{0})$ and not
$fin C^{2}(mathbb{R}^{n})$. Otherwise, if $t>0$,
$$
frac{f(te_{i})-f(0)}{t}=frac{tfleft( e_{i}right) }{t}=f(e_{i}),
$$

while if $t<0$,
$$
frac{f(te_{i})-f(0)}{t}=-frac{tfleft( -e_{i}right) }{t}=-f(-e_{i}),
$$

and so if $frac{partial f}{partial x_{i}}(0)$ exists, then necessarily,
$f(e_{i})=-f(-e_{i})$, which is only possible if $f(e_{i})=f(-e_{i})=0$, since
$fgeq0$. But then by convexity, writing
$$
x=frac{1}{n}sum_{i=1}^{n}nx_{i}e_{i}%
$$

you get
$$
0leq f(x)leqfrac{1}{n}sum_{i=1}^{n}f(nx_{i}e_{i})=frac{1}{n}sum
_{i=1}^{n}n|x_{i}|f((operatorname*{sgn}x_{i})e_{i})=0,
$$

which implies that $f=0$.



So LinAlg's conjecture is true.



Note also that if a function $fin C^{1}(mathbb{R}^{n}setminus{0})$ is
positively homogeneous of degree one, then its partial derivatives
$frac{partial f}{partial x_{i}}$ are positively homogeneous of degree zero,
so $frac{partial f}{partial x_{i}}(tx)=frac{partial f}{partial x_{i}%
}(x)$
for all $t>0$.






share|cite|improve this answer









$endgroup$



I think you need to assume $fin C^{2}(mathbb{R}^{n}setminus{0})$ and not
$fin C^{2}(mathbb{R}^{n})$. Otherwise, if $t>0$,
$$
frac{f(te_{i})-f(0)}{t}=frac{tfleft( e_{i}right) }{t}=f(e_{i}),
$$

while if $t<0$,
$$
frac{f(te_{i})-f(0)}{t}=-frac{tfleft( -e_{i}right) }{t}=-f(-e_{i}),
$$

and so if $frac{partial f}{partial x_{i}}(0)$ exists, then necessarily,
$f(e_{i})=-f(-e_{i})$, which is only possible if $f(e_{i})=f(-e_{i})=0$, since
$fgeq0$. But then by convexity, writing
$$
x=frac{1}{n}sum_{i=1}^{n}nx_{i}e_{i}%
$$

you get
$$
0leq f(x)leqfrac{1}{n}sum_{i=1}^{n}f(nx_{i}e_{i})=frac{1}{n}sum
_{i=1}^{n}n|x_{i}|f((operatorname*{sgn}x_{i})e_{i})=0,
$$

which implies that $f=0$.



So LinAlg's conjecture is true.



Note also that if a function $fin C^{1}(mathbb{R}^{n}setminus{0})$ is
positively homogeneous of degree one, then its partial derivatives
$frac{partial f}{partial x_{i}}$ are positively homogeneous of degree zero,
so $frac{partial f}{partial x_{i}}(tx)=frac{partial f}{partial x_{i}%
}(x)$
for all $t>0$.







share|cite|improve this answer












share|cite|improve this answer



share|cite|improve this answer










answered Jan 6 at 19:51









Gio67Gio67

12.5k1626




12.5k1626












  • $begingroup$
    Thank you for proving my conjecture.
    $endgroup$
    – LinAlg
    Jan 6 at 20:03










  • $begingroup$
    Thanks for your interesting post, providing a proof of LinAlg's conjecture. I hope you do not mind, I will accept his answer and give him the bounty. (Un)fortunately thanks to his answer (as well as your post) I realized my question is ill-posed and I need to review everything. Thank you very much for your time and help. +1, of course.
    $endgroup$
    – Romeo
    Jan 7 at 12:38










  • $begingroup$
    Thanks to what you wrote in the first line, I realized it is still meaningful to formulate the same question asking only $f in C^2(mathbb R^n setminus {0})$ (and condition (B) holding for every $x ne 0$). Suppose f is not identically vanishing, too. Is it true that the set $E_1$ does not contain straight segments iff (B) holds? Thank you very much again.
    $endgroup$
    – Romeo
    Jan 10 at 9:39




















  • $begingroup$
    Thank you for proving my conjecture.
    $endgroup$
    – LinAlg
    Jan 6 at 20:03










  • $begingroup$
    Thanks for your interesting post, providing a proof of LinAlg's conjecture. I hope you do not mind, I will accept his answer and give him the bounty. (Un)fortunately thanks to his answer (as well as your post) I realized my question is ill-posed and I need to review everything. Thank you very much for your time and help. +1, of course.
    $endgroup$
    – Romeo
    Jan 7 at 12:38










  • $begingroup$
    Thanks to what you wrote in the first line, I realized it is still meaningful to formulate the same question asking only $f in C^2(mathbb R^n setminus {0})$ (and condition (B) holding for every $x ne 0$). Suppose f is not identically vanishing, too. Is it true that the set $E_1$ does not contain straight segments iff (B) holds? Thank you very much again.
    $endgroup$
    – Romeo
    Jan 10 at 9:39


















$begingroup$
Thank you for proving my conjecture.
$endgroup$
– LinAlg
Jan 6 at 20:03




$begingroup$
Thank you for proving my conjecture.
$endgroup$
– LinAlg
Jan 6 at 20:03












$begingroup$
Thanks for your interesting post, providing a proof of LinAlg's conjecture. I hope you do not mind, I will accept his answer and give him the bounty. (Un)fortunately thanks to his answer (as well as your post) I realized my question is ill-posed and I need to review everything. Thank you very much for your time and help. +1, of course.
$endgroup$
– Romeo
Jan 7 at 12:38




$begingroup$
Thanks for your interesting post, providing a proof of LinAlg's conjecture. I hope you do not mind, I will accept his answer and give him the bounty. (Un)fortunately thanks to his answer (as well as your post) I realized my question is ill-posed and I need to review everything. Thank you very much for your time and help. +1, of course.
$endgroup$
– Romeo
Jan 7 at 12:38












$begingroup$
Thanks to what you wrote in the first line, I realized it is still meaningful to formulate the same question asking only $f in C^2(mathbb R^n setminus {0})$ (and condition (B) holding for every $x ne 0$). Suppose f is not identically vanishing, too. Is it true that the set $E_1$ does not contain straight segments iff (B) holds? Thank you very much again.
$endgroup$
– Romeo
Jan 10 at 9:39






$begingroup$
Thanks to what you wrote in the first line, I realized it is still meaningful to formulate the same question asking only $f in C^2(mathbb R^n setminus {0})$ (and condition (B) holding for every $x ne 0$). Suppose f is not identically vanishing, too. Is it true that the set $E_1$ does not contain straight segments iff (B) holds? Thank you very much again.
$endgroup$
– Romeo
Jan 10 at 9:39




















draft saved

draft discarded




















































Thanks for contributing an answer to Mathematics Stack Exchange!


  • Please be sure to answer the question. Provide details and share your research!

But avoid



  • Asking for help, clarification, or responding to other answers.

  • Making statements based on opinion; back them up with references or personal experience.


Use MathJax to format equations. MathJax reference.


To learn more, see our tips on writing great answers.




draft saved


draft discarded














StackExchange.ready(
function () {
StackExchange.openid.initPostLogin('.new-post-login', 'https%3a%2f%2fmath.stackexchange.com%2fquestions%2f3056984%2fstrict-convexity-and-equivalent-conditions%23new-answer', 'question_page');
}
);

Post as a guest















Required, but never shown





















































Required, but never shown














Required, but never shown












Required, but never shown







Required, but never shown

































Required, but never shown














Required, but never shown












Required, but never shown







Required, but never shown







Popular posts from this blog

Human spaceflight

Can not write log (Is /dev/pts mounted?) - openpty in Ubuntu-on-Windows?

File:DeusFollowingSea.jpg